8
$\begingroup$

In the Cohen-Tannoudji Quantum Physics book, Complement BII, says:

[...] two operators $A$ and $B$ with both commute with their commutator. An argument modeled on the preceding one shows that, if we have:

\begin{align} [A,C]=[B,C]=0 \end{align} with $C=[A,B]$, then: \begin{align} [A,F(B)]=[A,B]F'(B) \end{align}

Then this last property, is used to proof Glauber's Formula. \begin{align*} e^Ae^B=e^{A+B}e^{\frac{1}{2}[A,B]} \end{align*}

I understand this proof.

But I couldn't find a way to demonstrate, \begin{align} [A,C]=[B,C]=0 \end{align} with $C=[A,B]$, then: \begin{align} [A,F(B)]=[A,B]F'(B). \end{align}

I would like to know to do this, so I can understand better the Glauber's Formula proof.

$\endgroup$
4
  • 1
    $\begingroup$ Someone correct me, but it seems to me, that this is an assumption for the proof, rather than a general statement about arbitrary operators. $\endgroup$
    – CuriousOne
    Aug 30, 2014 at 5:08
  • 1
    $\begingroup$ @CuriousOne I think Ignacio may have worded this slightly ambiguously: $[A,\,[A,\,B]]=[B,\,[A,\,B]]=0$ is an assumption, and the proof Ignacio seeks is $[A,\,[A,\,B]]=[B,\,[A,\,B]]=0 \Rightarrow [A,\,F(B)]=[A,\,B]\,F(B)$ (given analyticity assumptions on $F$ and an assumption that the domains of $A,\,B,\,C$ are restricted to make the analyticity assumptions work). Is this a sound way of putting your question, Ignacio? $\endgroup$ Aug 30, 2014 at 7:39
  • $\begingroup$ @WetSavannaAnimalakaRodVance: Thanks! Now I get it. $\endgroup$
    – CuriousOne
    Aug 30, 2014 at 7:58
  • $\begingroup$ For the truncated BCH formula, see also physics.stackexchange.com/q/132886/2451 and links therein. $\endgroup$
    – Qmechanic
    Nov 20, 2015 at 13:45

3 Answers 3

6
$\begingroup$

A standard thing done in proofs of identities involving commutators is to expand things in Taylor series. Since $F(B) = \sum_{n=0}^\infty f_n B^n$, we have $$ [A, F(B)] = \sum_{n=0}^\infty f_n (AB^n - B^nA) $$ (commutators distribute over sums, as you can check). Then take one of the parenthesized terms, say $B^nA$, and move the $A$ through to the other side of the $B$'s, one at a time. Each time picks up another term with a $C$ and $n-1$ $B$'s, which you are free to arrange however you want, because $[B,C] = 0$. You should find your $AB^n$ terms cancel, leaving $$ [A, f(B)] = \sum_{n=0}^\infty f_n n B^{n-1} C, $$ where the $C$ can be put anywhere, including outside the sum. But the sum is just the Taylor series for $F'(B)$.

$\endgroup$
1
$\begingroup$

Here I will give some algebra method for the proof of Glauber's formula:

Assume $F(t)=e^{At} e^{Bt}$ : $$ \dfrac{d}{d t} F(t) = A e^{A t} e^{B t} + e^{A t} B e^{B t} = (A+e^{A t}Be^{-At} ) F(t) \tag{1} $$ Recall that the Hadamard's lemma (Proved in appendix): $$ \boxed{e^{A t} B e^{-A t} = B+ t[A,B]+\dfrac{t^2}{2!}[A,[A,B]]+\dfrac{t^3}{3!}[A,[A,[A,B]]]+\cdots} \tag{2} $$ then the equation $(2)$ can be simplified as ($[A,B]=Constant$) : $$ \dfrac{d}{d t} F(t) = (A+B+t[A,B])F(t) \tag{3} $$ Assume $G(t) = e^{At+Bt+f(t)H(A,B)}$ : $$ \dfrac{d}{d t}G(t) = (A+B+f'(t)H(A,B))G(t) \tag{4} $$ Let $\dfrac{d}{d t}F(t) = \dfrac{d}{d t}G(t)$ : \begin{align} f'(t) = t \Rightarrow f(t) & = \dfrac{1}{2}t^2+C \tag{5} \\ H(A,B) & = [A,B] \tag{6} \\ F(t) & = G(t) \tag{7} \end{align} $$ (5)\&(6)\&(7) \quad \Rightarrow \quad e^{A t}e^{B t} = e^{A t+B t+(\dfrac{1}{2}t^2+C)[A,B]} $$

  • $t=0 \Rightarrow C=0$;
  • $t=1 \Rightarrow $ $$\boxed{e^A e^B=e^{A+B+\dfrac{1}{2}[A,B]}}$$

Appendix for Hadamard's lemma: $$ \boxed{e^{A t} B e^{-A t} = B+ t[A,B]+\dfrac{t^2}{2!}[A,[A,B]]+\dfrac{t^3}{3!}[A,[A,[A,B]]]+\cdots} $$ Assume $Y(t) =e^{A t} B e^{-At} $ \begin{align} Y^{(1)}(t) & = e^{At}(AB-BA)e^{-At} = e^{At}[A,B]e^{-At} \\ Y^{(2)}(t) & = e^{At} A [A,B]-[A,B] A e^{-At} = e^{At}[A,[A,B]]e^{-At} \\ Y^{(3)}(t) & = e^{At} [A,[A,[A,B]]] e^{-At} \\ & \cdots\cdots \nonumber \end{align} $$ \Rightarrow \quad Y(t) = \sum_{n=0}^{\infty} \dfrac{t^n}{n!}Y^{(n)}(t)|_{t=0}=B+t[A,B]+\dfrac{t^2}{2!}[A,[A,B]]+\dfrac{t^3}{3!}[A,[A,[A,B]]]+\cdots $$

$\endgroup$
0
$\begingroup$

I think $[A,C]=[B,C]=0$ with $C=[A,B]$ is an assumption, because there exist counterexamples: for $A=\sigma_x$ are $B=\sigma_y$ Pauli matrix along $x, y$ directions respectively. then $C=2i\sigma_z$ is Pauli matrix along z direction. Obviously $[A,C]\neq 0$ and $[B,C]\neq 0$.

$\endgroup$
1
  • $\begingroup$ It is definitely an assumption. I don't think the questioner has a problem with it being an assumption. He clearly states that he fails to see how this assumption implies $[A,F(B)]=[A,B]F'(B)$. $\endgroup$
    – Martin
    Nov 20, 2015 at 14:05

Your Answer

By clicking “Post Your Answer”, you agree to our terms of service and acknowledge you have read our privacy policy.

Not the answer you're looking for? Browse other questions tagged or ask your own question.